Osteopathic Orthopedic Surgery Residency Reviews

This forum made possible through the generous support of SDN members, donors, and sponsors. Thank you.

Wozo30

Full Member
10+ Year Member
Joined
Nov 21, 2008
Messages
51
Reaction score
8
As a recent matcher into DO ortho I know how difficult it was to find information on various programs and had many questions as to which programs favored rotators and which focused primarily on boards. There are other websites that have also posted reviews but over time many of these have become dated. I plan to follow this thread through residency and beyond and am willing to help in any way that I can. I hope others follow suit and continue to provide up to date information for students interested in ortho. If anyone wishes to post experiences, good or bad, but fear "being found out.". Feel free to PM me and I will post it anonymously.

Members don't see this ad.
 
  • Like
Reactions: 2 users
Much of this information may have been posted elsewhere in various threads but my goal is to form a central location that can be stickied for students to find easily and compare between programs.

Program:

General:

Attendings:

Residents:

Didactics:

Operative Experience:

Clinic Experience:

Research:

Lifestyle:

Pros:

Cons:
 
Much of this information may have been posted elsewhere in various threads but my goal is to form a central location that can be stickied for students to find easily and compare between programs.

Program:Riverside, CA

General: location is a big plus here as it is close to LA and a small drive to Phoenix, Las Vegas and San Diego. They take 3 residents per year and is historically a rotation that students get worked to death 100+ hrs per week.

Attendings: Dr. Faerber is the PD and is sports med trained. He is and interesting character as sometimes he can be pleasant while others he is screaming at the OR staff, but seems to be a fair guy. Dr. Schlecter is the peds guy and is in charge of education. He does an excellent job and in enjoyable during didactics. He can be rough as he has a sarcastic humor and no patience but his goal is to make you better. Dr. Capital is the hand surgeon and is quiet but very nice. The others I had little experience with outside of clinic.

Residents: A good mix of guys that seemed very well educated, I think one struggled. The seniors were the "smartest" I had been around, though whether this was true or not who knows. They just talked a lot and quoted a bunch of literature. But they all did very well on OITE.

Didactics: This was done every morning. I felt they were weak outside of Dr. Schlecter's days. Sometimes watched videos from orthobullets, or did questions. Not much in terms of fracture conference but the PD was out of town a couple of the Fridays they were normally done.

Operative Experience: Touted as their strong suit they residents do get a lot of OR time. The intern does a lot of the day consults and PGY2 is responsible for day to day as well (clinic, consults, help in OR). The PGY3 seems to be the turning point as many times they ran their own OR. PGY4 you do joints and sports at Kaiser. PGY5 do more of the difficult trauma and instruction to the juniors. Unfortunately much of what they do, 80%, is either trauma or infection control. Best peds experience throughout training.

Clinic Experience: Lots of clinic as a student, 4 days per week. Lots of autonomy though. We were able to see patients on our own, present to the attending or senior then explain the treatment plan to the patient. Lots of note writing and decision making make this an excellent experience. Many patients speak Spanish.

Research: Like most DO programs the response when asked about research is, "there are opportunities available". In general I think the bulk of work is done by the residents and I am not aware of a "stats department" to assist them.

Lifestyle: Intern year seemed busy, PGY2 seemed horrible but lifestyle improved thereafter. As a PGY2 the day is split into two 12 hour shifts with the third PGY2 taking spine service. This helps tremendously with call as a senior. Lifestyle not so good as student. Expected to be there even when it is not busy and get a lot of soft goodbyes. "you can go if you want...or you can stay."

Pros: Lots of autonomy and operative experience early on. Best trauma and peds experience associated with the general residency rather than doing out rotations

Cons: Too much autonomy. Many times it's seniors teaching juniors without much attending oversight. Too many opportunities to makes mistakes. Very little joint and sports experience which was evident during these procedures.
 
Members don't see this ad :)
Program: St. Mary's Blue Springs, MO

General: Have taken anywhere from 2-4 residents in the past but are supposedly going to be locked in at 3 residents per year. Most surgery is done at North Kansas City hospital. KC is a great city with nice people and good cost of living.

Attendings: Dr. Orth the PD is a very nice doc who has the best interests of the residents in mind. Aside from the addition of two new sports med docs, the attendings are all general ortho. Thus this is a very good general ortho residency. Two of the docs do a lot of primary total joints. One doc is reluctant to allow residents participate in surgery and thus many seniors do not operate with him.

Residents: Best all around guys I was around. Most had a very good sense of humor and joked around a lot. The two seniors are both going into sports med. 2 of last years senior did sports med, and one foot and ankle.

Didactics: Kind of just thrown together. Probably the poorest of any place I had been to. Many of the attendings do not like the residents to go to didactics if it cuts into their time in the OR. None of the atendings have PAs and rely on the residents. I think they are trying to improve this portion of the residency by making a more coherent schedule, but remains to be seen.

Operative Experience: Varies depending on which attending you are with. Some will let the senior do the whole case (totals, ACL, scope, etc..) without being present, and others will let the resident close on a TKA. Very little trauma and no peds outside of general ortho. Good mix of general ortho but little hand, no foot and ankle. Hand with another doc in the area, peds at Children's Mercy and Onc with Dr. Rosenthal in KC. Trauma is 3 months in Las Vegas during the PGY4 year.

Clinic Experience: Kind of a drag of the residency. As there are no "hand, sports, joint" docs, the residents finish whatever they are doing for that day then head to clinic to help out. At times there can be 6-7 residents in clinic at a time. Also some physicians are inefficient and routinely run 1-2 even 3 hours behind. Much of this comes from overscheduling of surgeries. As a student you basically tail residents or the attending and just listen.

Research: Didnt see much going on. There was talk of getting access to the KCUMB anatomy lab but mentioned that this has been ongoing for the last couple of years. But again no "stats team".

Lifestyle: Probably the best lifestyle of all the places I rotated. Days start early~5am but many times finished by 5pm. Call is pretty relaxed as NKC and St. Mary's doesnt seem to get much trauma.

Pros: Excellent general orthopedics

Cons: operative experience varies greatly depending on which attending you are with
 
Program: Doctors Hospital Columbus, OH

General: Columbus is a medium sized city without pro baseball or football but close to Cincinnati and Cleveland. Interviewed 37 for 5 spots this year.

Attendings: Too many to go through. Dr. Baker is the PD but I did not even get to meet him until the interview as 3 of 4 weeks is spent at Grant hospital. The trauma team at Grant is excellent. Many of the attendings did not speak to me but as this is a busy audition site, many times there were 3-4 students in each procedure.

Residents: 25!! Lots of peeps here with different personalities. Very friendly as all met me with a handshake and hello. Even though 80+ students had come through many recognized me at the interview. Lots get placed in very good fellowships.

Didactics: Some students really liked their didactics, but I did not. One day a week for 6hrs. It was a variable day where the interns got slaughtered at fracture conference by the trauma attendings, a few presentations by residents on different topics, and some sort of learning module such as casting. But once a week wasnt enough personally. Plus with everyone in the room at times there were 40+!!

Operative Experience: Good exposure to various procedures that you did not get to see elsewhere, especially the trauma. For residents though I saw many juniors getting bumped by seniors. On a simple knee scope I thought the resident was a second year due to his struggling through the procedure, but turns out he was the chief. I saw something similar with another chief during a fibula fracture. A common theme is too many residents in each case, especially trauma as there are also 3 fellows to compete with.

Clinic Experience: Outside of hand and sports med weeks I saw little clinic. Some of the residents had mentioned to me that they think they have too much clinic. Hand clinic was awesome as I saw what true efficiency is like.

Research: Probably the closest to an academic DO residency that I saw. Many of the trauma attendings as well as peds attendings have ongoing research projects that residents can "jump on" I think they may also have staff to assist with chart review and such at Grants, but this I am not sure of.

Lifestyle: Very predictable due to scheduling. If you are on a service that is all you do (i.e. if you are on hand for two weeks, you do hand clinic and OR...no going to help finish out others clinic.) This makes it nice to focus your attention as well as schedule your family life.

Pros: Good all around residency that has excellent attendings and opportunities for research

Cons: Operative skills of residents was suspect. This may be due to the individual and not the training, but 2/4 chiefs I saw struggling and only operated with 3 of them.
 
Program: St. Anthony's Oklahoma City, OK

General: OKC is a growing city with a pro ball team and people are salt of the earth. Take 2 residents per year.

Attendings: Dr. West is the PD and is a very laid back and down to earth guy. Dr. Reddick and Dr. Tkach are the total joint guys, Reddick is a rough neck and expects a lot from his residents but is an excellent teacher. Tkach is one of the best joint guys in OK and has patients travel from 4 states to come see him. Obrien and Ewing are the hand guys that give great instruction in the OR. Also time can be spent with the other docs not associated with the residency if time is available.

Residents: Small group of guys that work hard. The new chiefs are very nice guys that will continue to improve the residency.

Didactics: Some students didnt like their didactics but I thought they were great. A little too much "slide reading" but an hour or two each morning with fracture conference on wed. At least two attendings were present at fracture conference and brought good perspective. Many of topics covered helped answer pimp questions throughout the months so I felt were very relevant.

Operative Experience: Best I saw anywhere I went, 4.5 days a week. On joints service will do at least 5/day perhaps more, Obrien routinely does 16 to sometimes 20 cases during his OR days. Chiefs do all trauma cases the day after they come in. Many times chiefs either did the case or taught a junior with attending supervision. Graduate with 3500+ cases.

Clinic Experience: Obviously very little with just 0.5 days a week though more time is spent in clinic as an intern. I dont believe there is any set "resident clinic", which for me was a huge plus as being a proficient surgeon is my biggest concern.

Research: Again not much going on outside of residents own projects.

Lifestyle: Difficult at times. Especially call weekends where St. Anthony's is taking trauma 2 call. With only 10 residents there are more call shifts than would be elsewhere.

Pros: Tons of OR experience with graduating autonomy. All the attendings are incredibly nice and willing to teach (honestly not uncommon to have attendings ignore you exist at other places.)

Cons: OKC may not be a big enough city for some, brutal call shifts.
 
Anonymous

Program: St. John Oakland

General: Base hospital in Madison Heights, which is real close to Royal Oak (bars/restaurants). Take 2 residents a year. Board scores highly emphasized. Interviewed 575+ but really want 600+ preferably higher. Invite non rotators because they don't get enough rotators. If they like you at the interview, then they'll call you up to come for a few days before they make an offer to you. Real laid back, no malignant personalities.

Attendings: Three attendings at the base hospital. Najarian, Kelly, and Hill. Najarian is the director, old school, but a very nice guy. Pimps a little, care a lot about boards and how you get along with his residents. Treats his residents like his sons (and a daughter). The other two are not as involved but residents scrub with them. Both are nice guys.

Residents: Really laid back guys but really smart. Education is a priority here so they are well read. Take 2 residents a year. These guys get along well and are really down to earth, no malignant personalities.

Didactics: Strong. OITE reviews, Journal club. Fracture Conf.

Operative Experience: Weak at the base hospital as there not a lot of cases to go around. However, their out rotations are solid, go to Sinai Grace/DMC. Tyler, TX for Trauma. Go to other big hospitals in St. John system. While at the base hospital, lots of autonomy as Naj lets the senior run their own rooms, who in turn teach juniors.

Clinic Experience: Pretty good, as a PGY2, you go to Naj's office 2-3 times a week. Mostly general ortho stuff.

Research: They didn't talk about this much while I was there so not sure.

Lifestyle: Very chill, start around 630 most days at the base hospital. Not sure about other hospitals they visit for outrotations

Pros: Smart guys, family like atmosphere here, very laid back, no malignant personalities, education is the top priority, you don't get treated like a glorified PA here, they really get you involved early on in the OR as a junior resident

Cons: Case load at the base hospital is low, but don't be deterred, Chiefs had great operating skills as they have some good outrotations that provides solid number of cases.
 
I don't know how you feel about pulling info from the other well known ortho site but they have a good amount of residency reviews there. I don't see an issue with it though.
 
I know most of my stuff is the same, but id rather not. It seems they were touchy even mentioning the name of other sites so better if we didn't pull the info I suppose
 
Anonymous

Program: South Pointe Hospital

General: Located outside of Cleveland. Very relaxed program. Just got a new PD, Dr. Hampton. Took 3 residents this year. Spend time at South Pointe/HillCrest/ and CCF main. Pretty nice hospital with friendly staff. Trauma in Denver, Tumor/peds at CCF main. Not heavy on boards and consider sub 500 scores.

Attendings: 3 attendings at the base hospital. Hampton, new PD, nice guy, pimps a little. Krahe, the old director, awesome guy. Masin, good guy. Mackel is also involved but not as heavily as the other 3. All general guys, very nice and laid back.

Residents: Very laid back, nice guys, but very guarded. One of the chiefs here was a bit uptight and he needs to relax. Rest of the crew was very easy to get along with.

Didactics: Strong, daily morning lectures, Fx conf, monthly didactics at CCF main.

Operative Experience: Weak at the base hosiptal, they graduate with fairly low number of cases here, however, they get autonomy in the OR as attendings don't scrub for the most part.

Clinic Experience: Strong, ton of clinic, most out of any progam I checked out.

Research: Ample, with their association with CCF, you can really get involved in some cool research projects.

Lifestyle: Really chill, start 630, no in house call, very relaxed, go home at a very reasonable time.

Pros: Cleveland clinic affiliation, lots of autonomy in the OR, strong didactics.

Cons: Very guarded and not upfront with their resident selection/interview process. Cleveland is pretty awful.
 
Anonymous

Program: Broward Health, Ft Lauderdale, FL

General: Level 1 trauma center in downtown Ft. Lauderdale. Every specialty in house except for peds. Take two residents a year, getting heavy on board scores. One of the few programs that like high class rank and are verbal about it.

Attendings: Dr. Rush is the director, doesn't operate much at Broward but is well connected so is the program director. Pretty malignant to his residents and students and thinks pretty highly of himself IMO. Dr. Cross is the associate director, trauma guy, very nice, he is much more involved with the residency than Dr. Rush and should be the director. Dr. Wells, sports guy, very heavily involved with residents as well, nice guy. Kapila is hands, smart guy, but has no business taking trauma call (put a side plate on a reverse obliquity!) Carriera is Foot/Ankle, big name in SoFlo, very nice and really smart. Roth is another Trauma guy, nice guy and is pretty good.

Residents: Take 2 residents a year, nice guys, no malignant personalities. One of the PGY3 is solid, one of the best I saw anywhere. Chief here was a great dude. Two girls in the program and they just matched another one.

Didactics: Poor outside of Fracture conference where anywhere from 5 to 10 attendings would show up routinely. Best Fx conference out of any program I checked out.

Operative Experience: Solid from what I heard from the residents, but autonomy was an issue. Most of the attendings here seemed paranoid due to medical-legal environment of South Florida and didn't give residents too much autonomy. There was a lot of watching invloved, even the chief watched at times.

Clinic Experience: Resident run clinic, pretty solid, residents saw patient, made plans with attending to back them up

Research: From what I understand, they are trying to get more involved in research here. Ample for residents, depending on how much you want to be involved.

Lifestyle: Pretty hardcore, Level 1 trauma center, q3-4 call as a junior resident which is in house. You stay up when on call as it's pretty busy.

Pros: Location, very nice, lots of good stuff as it's level 1 trauma, every specialty in house with peds at MCG in Augusta, GA.

Cons: Lifestyle was pretty awful for residents. Program director is not very friendly/approachable and is not very involved at Broward.
 
Anonymous

Program: St. Vincent Mercy

General: Located in the @SShole of America per PD haha. Strong program, level 1 trauma center. Take 3 residents a year, really high on boards. Don't bother unless you at least have 620+. Took three guys this year, with two being 700+ so you can see where the emphasis is. 4th year is spent outside and is full of elective rotations.

Attendings: Dr. Jaeblon, the PD, old school guy, treats his residents like his sons. Tells you like it is. Weber, the new trauma guy, also very good. These are the only two attendings that are heavily involved with the residency.

Residents: Nice guys but very smart. Work really hard. Very laid back, well read.

Didactics: Very strong, strongest out of any program I saw. OITE questions every morning, Journal club weekly, morning lectures by the PD.

Operative Experience: Excellent, PD doesn't even scrub, its a senior and a Junior. They don't have specialty based rotations, instead only have general ortho service, so lots of trauma.

Clinic Experience: Pretty good, monday morning is spent in clinic where residents see, patients, formulate their own plans, and bok their own cases.

Research: Not sure, they didn't talk about it much

Lifestyle: Awful, they work very hard here. In house call, show up very early, I showed up as early as 315 as a student, just ridiculous.

Pros: Strong didactics, great operating experience, laid back program, strong reputation.

Cons: Terrible location and terrible lifestyle
 
Members don't see this ad :)
As a recent matcher into DO ortho I know how difficult it was to find information on various programs and had many questions as to which programs favored rotators and which focused primarily on boards. There are other websites that have also posted reviews but over time many of these have become dated. I plan to follow this thread through residency and beyond and am willing to help in any way that I can. I hope others follow suit and continue to provide up to date information for students interested in ortho. If anyone wishes to post experiences, good or bad, but fear "being found out.". Feel free to PM me and I will post it anonymously.

awesome. thank you!
 
Hey, which places did u rotate in 4th year and which ones did u like and y?
thanks
 
Doctors, Riverside, OKC and KC. Read my reviews above for more details
 
Doctors Hospital, Columbus, OH
I felt that the upper level residents at the Doctors program that I have seen operate were impressive. Dr. Baker seems to trust the residents to perform entire cases and is able to provide teaching rather than just taking over the case. I think that the trauma training they receive at Grant makes them feel very comfortable handling all types of fracture work, even complicated intraarticular cases. Most of their training is from fellowship trained attendings with focused practices. Most of the graduates seem to do some type of fellowship. I do agree that the 1's and 2's do not do as much operating early on as the small community programs, but I think this is more than offset by the exposure to high level unique specialty cases. My experience has been that the programs where residents are doing cases early on is because the cases are boring to the older residents and are typical bread and butter general ortho cases. The 1's and 2's do most of the floor and clinic work which makes 3-5 year almost purely operative. That being said I have seen 2's do entire cases such as knee scopes, tibial nails, and carpal tunnels. Even though they have 5 spots per year, tons of students rotate through, so the odds of getting a spot are still slim. During auditions students get to do a different specialty each week such as Peds at the childrens hospital in Columbus, Level I trauma at Grant, outpt services like sports, hand, and spine. You will want to have over 600 on boards and try to meet as many residents as possible during your rotation to give yourself the best chance. This can be difficult because there are 25 of them. Columbus, Ohio seems to be an ideal place to do residency. With the university comes plenty of young cultured people, fun places to go out, and an affordable cost of living. Comparing DO residencies, it has to be near the top.
 
Program: Pinnacle Health (Harrisburg, PA)

General:
Located in and around the Harrisburg, PA area. There is a great mix of settings including a downtown more "urban" hospital, a community hospital with high-volume (with an attached surgical center), a brand new hospital opening summer of 2014, and finally a strong affiliation with Penn State (Foot & Ankle, Trauma, Peds) and Children's in ATL (Peds). Very nice mid-sized community with its fair share of nice areas as well as some that are less desirable. However, these "less desirable" areas are often necessary to provide some of the patient population that we as future Orthopedic Surgeons should seek out to further our surgical skills and exposure. If you are looking for big-time culture and "city" type of things then this may not be the program for you, but if you want to come and work hard and learn your craft at the highest level this is the right program.

Attendings:
Great mix of attendings. Attendings in this program represent the spectrum of different personality types but all are really engaged in resident learning and are great in the OR. One of the best elements of the program in my opinion. Attendings here are very "hands-off" in the OR, and will allow you as a resident to "struggle" a little and think things out on your own before intervening. This is essential in the residency training process because at the end 5 years you need to be able to do this on your own, so finding attendings that will teach but allow you to use your own judgment and develop your surgical confidence is a necessity. Almost all sub-specialties (with the exception of Foot & Ankle, which is done at Penn State about 20mins away) are represented. Program director and assistant director Dr. Ackerman and Dr. DeLuca are fantastic. Great surgeons who want to be a part of the residency education process. Attendings here take care of their residents and a few times a year hold outings for residents, and if you are a big Penn State fan there is potential to get Penn State football tickets as many of the attendings are big fans or alums of the school.

Residents:
BEST RESIDENTS OF ANY PROGRAM! I say this without any reservation. From top-to-bottom this program had the best residents that I worked with in my months of Ortho auditions. All of the residents really knew their stuff, were great with the patients, had surgical skills exceeding their level of training (as compared to other programs), were great teachers in and out of the OR, and above all were all REALLY COOL PEOPLE! Don't underestimate how important it is to go to a program where the residents you will be working with are guys/girls that you want to hang out with everyday. I found this to be the case at Harrisburg. All residents and resident classes seem to have a great relationship with one another. This is a program where the guys will go out and have a beer together and believe it or not, that says a lot about the team here because elsewhere this was not the case. There is never any question about back-up as anyone that needs help with anything from H&Ps to reductions in the ER has whomever they need without ever having to ask twice. Residents here are the BEST FEATURE OF THIS PROGRAM.

Didactics:
Very strong didactics with something educational every day. Best fracture conference of any program I have been too. The fracture conference here is a great learning tool because it becomes a discussion between the resident presenting the case and the attendings which often becomes a discussion between the attendings for the residents benefit. It is quite different from other places where it becomes a giant "pimping" session. Each attending will discuss their approach to the fracture and what they would have done and why. As a student this was really great to see because as much as I could study I would never have the experience or knowledge of the attendings in the room so I always wanted to hear what they thought and more importantly their answers to the "why" questions. This was the only program that truly had that as part of the curriculum. Regular OITE, OKU, Journal Article, etc. review on a daily basis. As a junior resident in this program expect to be on your toes everyday as the seniors direct the majority of their pimping" towards the juniors which is great to have as a junior because it accelerates your education. As a student I can guarantee you will learn a tremendous amount here. I think about my Ortho knowledge before and after this rotation and it is remarkably different. As someone who read throughout the whole audition season I still managed to learn so much here on service which was great as a student. Also, know that the other students who are coming here are smart and know their stuff so if you want to stand-out be prepared and really know your OR cases.

Operative Experience:
You will see and do just about everything in this program. Tons of primary joints, sports cases, fractures, trauma, spine, etc. You will see and do everything multiple times and residents in this program have perhaps the highest case load of any MD/DO program in the country. That should say enough in and of itself. Bottom line, come here if you like to operate.

Clinic Experience:
Primarily resident run clinic run by a chief resident and attended by junior residents. There is some talk of expansion to include going to the attendings' offices to see private patients. I believe this is still in the works. This may be the only weak spot of the program in the eyes of some. My personal feeling is that it is much more important to develop your skills in the OR and as long as your knowledge of physical exam and office management of the Orthopedic patient is solid you will learn how to work in the office with more experience once you are a fellow/attending. If you beg to differ, please consider this when looking at different programs.

Research:
Small opportunities here or there. Potential to work with Penn State attendings on projects. Don't come here if you want to be a researcher, come here if you want to become a top-notch surgeon where nothing is going to phase you.

Lifestyle:
Great repoire between residents, low-cost of living in the area, WEGMANS (for this who know how important this is, there is one close by), great attendings, nice hospitals with beautiful Ortho floors, all contributes to a good lifestyle here. Depending which attendings you are covering some days you can be done relatively early and can leave the hospital as long as your service is tight. There is an expectation that you go home and study or do some reading though. But I did not hear of a single resident in the program that complained about their lifestyle.

Pros:
This whole review is pretty much a list of pro's for this program but if they were to be listed I would say the following are some of the biggest pros.
1. Residents
2. Operative case load
3. Quality attendings
4. Nice hospitals with lots of resources
5. Great out rotations
6. Low cost of living
7. Strong didactics

Cons:
For students, this is a highly selective program. Don't come here without having studied Handbook, Netters, and the big stuff in Millers (esp. Sports topics when working with the program director). You can look foolish here very fast if you don't know your stuff so come prepared. Also, think twice about coming here to rotate if you aren't prepared to work at a high level and operate A TON. I was told before rotating here that this was "Ortho Bootcamp" and it certainly was (NB: ALL OF THESE THINGS ARE PROS TO ME, how you feel about this is up to you, some people are looking for a lower-intensity program).
1. Clinic experience

During my summer/fall of Ortho auditions I consistently heard 3 programs listed as the top programs: Doctor's (Columbus), Grandview (Dayton), and Pinnacle Health (Harrisburg). Perhaps all three offer different strengths/weaknesses. I rotated at both Grandview and Pinnacle Health but I strongly felt that Pinnacle Health is very likely to be the TOP OSTEOPATHIC ORTHO program in the COUNTRY.

Hope this helps everyone. Good luck to future students on the Ortho trail!
 
Appreciate everyone posting their reviews of all the programs. Again with the match not being out yet but many 3rd years trying to decide where to rotate next year and trying to set those rotations up (3rd years start calling programs early in the year to hopefully get the schedule you want), please feel free to send me a private message and I will post your review as anonymous. Others reading these reviews I encourage you to send questions to myself and other posters to find more information.
 
St. Joseph Health Center - Warren, Ohio

The Ohio gem program

General/Attendings: This is a community program that covers two main hospitals. They cover St. Joseph's a 220 bed hospital where they learn their everything-under-the-sun of general cases, St. Elizabeth's which is a 550 bed level 1 trauma and tertiary care center, Akron's children peds hospital in youngstown, and 2-3 surgical centers. Their subspecialty exposure is extremely good where they train with almost all MD fellowship trained attendings in the area that range from Joints trained at New England Baptist, peds trained at Hospital for Special Surgery to a sports trained at Steadman-Hawkins. Their MD fellowship trained surgeons have connections, and in the past 7 years they've matched I think two into Cleveland Clinic for joints. Because of the great amount of subspecialty exposure from very well-trained attendings, they are not required to do any out rotations unless they want to for electives. Unlike some programs that boast high OR exposure, all the attendings here actually let the residents fly at an early stage. I saw an intern in his second week doing carpal tunnels and triggers fingers. I saw him later that week do a sliding hip screw with the proper technique taught by the chief.


Didactics: They have didactics a couple days in the morning and friday afternoon. It's the usual lectures, journal club, OITE, and other educational activities. The fracture conferences are always educational because they cover and operate at a level 1 trauma hospital. Unlike many of the programs you will come across aside from Doctor's hospital, the fracture conferences are so diverse and variable you will walk out of this program prepared for almost any fracture. The trauma conference has the trauma attendings heavily involved including surgeon's like Dr. Sontich who's residency trained at Cleveland clinic and trauma fellowship trained at Metrohealth ohio who works with the residents. Their didactic learning style is more do it yourself, which some may consider weaker.


Operative Experience: The operating experience is one of their greatest strengths. I got to rotate at 6 top ortho programs and can say that these guys were the best skilled surgeons I saw on the audition trail. In my first week at St. Joe's I saw a 4th year do 3 knee replacements, 2 hips, 2 ACL's (btb AND hamstring allograft), several knee and shoulder scopes all on his own. The attending was either not scrubbed in or was acting as his PA. Not only did he do them, but he did a great job with lots of confidence. I saw a chief put in pelvic screws under the direction of the traumatologist at st. E's. Overall, their chiefs were leagues ahead of the other residencies in terms of surgical training and confidence. These guys operate a lot and early. Other residencies boast about having a lot of operative experience, but St. Joe's had the residents actually doing all the cases instead of the attendings doing most of procedure.


Clinic: They previously had a resident run clinic and now are trying to get it back running with one of their new trauma guys that just came out of allegany gen for trauma. It should be up by 2014-2015. If you're looking for a really good clinic experience as a resident, I would look to other programs like Grandview.


Research: Of the many places that have research opportunities, St. Joe's surprisingly had one of the best amount of research for residents (aside from Columbus). They have their own statistician at St. E's that will help keep your project running. They also have affiliations with Youngstown State University where a couple of their residents have run their own ortho labs at the university. One of the former residents ran a biomechanics sports lab there during his residency before going to Allegheny general for sports. This one was of the biggest selling points for me because the future of orthopedics; Even if you are an excellent general surgeon, you'll often have to do a fellowship to market yourself in a city setting. Having research gives you an edge as a DO for better opportunities in five years regardless of your original plans starting as an intern.


Residents. They take 2 per year so they are a very tight knit group and just a bunch of great guys. They do hang outside of work. Very laid back and everyone takes care of each other. Very knowledgable and hardworking. This was one of the auditions were the ortho guys actually wanted to get to know the students and hung out with us when their was a light day.


Lifestyle was very balanced. They do take call at a level 1 trauma. Trauma rotation is expectedly a little more intensive. The residents as a whole seemed a lot happier than at the other places I went to. It seemed like there was a lot less BS time for the junior and senior residents having to act as retractor boys. They were either operating, quickly managing the floor, or out of the hospital reading/ enjoying life.


Location/Housing: Cheap housing. Relatively close to Cleveland, Pittsburgh, Erie, and Akron area. The surrounding areas are good to raise a family eg. Boardman/poland area.Youngstown is not the safest area. If you're a city person, this program will not suit you.


Med student rotation experience: If you don't come here for residency, you will learn more about fractures and how to describe them than at any place. They have their own med student fracture conference every week for the rotating students which makes you accountable to study from your handbook of fracture. When I was there, there were 10+ students so I didn't scrub in as much as other places. Although the residents get to do a lot, the operative experience for med students is weak. Taking call here as a med student is unreal, you will learn more about fractures and acute orthopedic injuries than any of the other community programs. It's a good atmosphere for a month with no malignant personalities. One of their base attendings asks a bunch of weirdly phrased ortho questions. You get a food stipend as well as a free pass to both Hospital's gyms. Highly suggest to everyone to rotate here if you can. I rotated with a bunch of med students throughout the audition trail, and everyone had nothing but great things to say about this program. Even if you don't match here, I guarantee this will tremendously improve your hand book of fracture knowledge by giving it context and knowledge on treatment.


Overall:
Pros:
1. Best surgically trained orthopedic residents, hands down
2. Amazing exposure with a ton of MD fellowship trained attendings who have connections to prestigious institutions
3. Excellent research opportunities with statistician and youngstown state university
4. All subspecialties are covered with no required out rotations
5.They have 4 months of electives their 4th year for additional training and fellowship opportunities
6. Strong trauma experience with coverage all 5 years
7. Tight-knit group of residents that were all extremely proud and happy to be in the program and had time to hang out with each other outside the hospital
8. This was also a big selling point for me; I was the happiest here.


Cons:
The didactics here are better than other programs, but you still have to be self motivated to read on your own here. They are more OR heavy than clinic as well. This is a very hands on program with a significant amount of surgical autonomy, which may not fit your training style if you like to learn in the OR by watching.They do not favor base students here, and heavily base their selection fairly on who worked the hardest, knew their ortho, and got along with the residents.

During the 2013 season, I rotated through St. Joe's, Doctors, Grandview, Toledo, Harrisburg, and 1 MD program. St. Joe's and Harrisburg were, in my humble opinion, the best orthopedic residencies for me. St. Joe's is the sleeper osteopathic orthopedic program that I didn't realize I'd like so much.
 
Getting close to that time of year again, good luck to all in the upcoming match. Please post with your reviews. Again if you feel like being discreet PM me and I will post it anonymously.

Program:

General:

Attendings:

Residents:

Didactics:

Operative Experience:

Clinic Experience:

Research:

Lifestyle:

Pros:

Cons:
 
Wozo30
Joined:
Nov 21, 2008
Messages:
40
Status:
Medical Student
Mercy St. Vincent Osteopathic Ortho

I checked my email this afternoon and found out I matched ortho in Toledo, OH. I am SOOOO stoked about this place I had to write my review because the previous review had some this wrong and I wanted to elaborate on a few points which I felt made this program unique for me.

Program: Mercy St. Vincent in Toledo, OH

General: Program is solid. Director is Jaeblon - has been PD for over 30 years. He tells it how it is and he makes sure his residents are prepared for anything after graduating. They take 3 a year and only choose from residents that rotate here, but the key is you have to have the board scores in order to even rotate (at least a 600). Residents get hands on during their first year and continue to do so as they move to 2nd, 3rd year and so on. This is the only osteo ortho program at a Level 1 trauma hospital.

Attendings: Residents book cases out of clinic with Jaeblon. The younger residents do the majority of the cases with the senior resident guiding them along the way. Dr. Jaeblon sits in the OR and lets the residents problem-solve through the case. The new trauma guy does a decent amount of cases (long bone fractures, pelvis & acetabulum, proximal humerus, nonunions). They have a fellowship trained spine surgeon who does some general as well. There are 2 fellowship trained sports guys (one guy just started a few months ago) who operate at different hospitals/surgery centers around town with the residents. 3 other general orthopods round out the main operating staff - 2 of them operate at other hospitals with resident coverage (students could not go to these hospitals). 3 Hand/Plastic surgeons take all hand call at St. V's and residents are a big part of those cases as well. 3 pediatric orthopods are located across town at another hospital where a pgy-3 spends 4 months (I felt this was a big plus since you did not have to leave town). One of the chief residents will be joining the pedipods after fellowship. In general, all the attendings teach the junior residents during cases and let them operate very early compared to other programs.

Residents: A big part of the program is senior residents teaching the junior residents. The 3 senior residents were all very well read (1 is doing a hand fellowship, 1 a total joints fellowship, and 1 a pediatrics fellowship). The senior residents run morning didactics and keep the juniors on their toes with pimping during cases. PGY-2s and 3s take the majority of the call with PGY-1s on for 8 shifts a month. PGY-5s take call from home and come in if needed. All of the residents get along well and like to have fun - they frequently get together outside the hospital. Residents also cover local high school sports on Fridays.

Didactics: Usually something every morning:
Mondays: Miller review and Topic lectures by Dr. jaeblon
Tuesdays: sports articles
Wednesdays: OITE review followed by a PGY-1 lecture
Thursdays: Hand lecture, Trauma conference, resident lecture, and articles with Dr. Jaeblon
Friday: Cadaver presentations
Usually a sawbones gathering or evening lecture at least once a month
Every resident presents their yearly paper in a power point lecture every month

Operative Experience: Very early operative experience with seniors teaching juniors. A lot of trauma cases in the summer - good amount of sports cases - spine cases 2 days a week (1 day at St. Vincents and usually another day at another hospital) - hand cases weekly. There are no assigned rotations - the residents divvy up the weeks cases in clinic every Monday (first case is a total hip, next case could be a spine, then Carpal tunnel, then a tibia nail that came into the trauma bay earlier that morning).

Clinic Experience: Mondays all day - clinic is clinic - 40-50 patients - residents book a lot of cases staffed by Dr. Jaeblon. Lots of total joints, shoulder and knee scopes, and hand cases.

Research: Not much of a presence right now. The new sports guy who just started is lining up research projects with the junior residents. He also is in charge of making sure the yearly paper requirements and power point presentations are complete. Most of the residents submit their papers to ortho journals.

Lifestyle: These guys are very busy, well read, and can handle any trauma that comes in the door. Residents work hard but also know how to have fun. Lots of traveling to different hospitals in town - 6 in Toledo and 1 in Defiance.

Pros: the biggest draw to this program is the 4th year. This year is spent on out rotations where residents set up rotations in areas they don't get a lot of (like foot/ankle and Tumor). THE ENITRE YEAR IS SPENT OUTSIDE TOLEDO!!!!!!!!! Residents usually rotate in areas that they want to do a fellowship to get exposure. Many of the residents go to places where family and friends live to do their rotations. A lot of the guys went to Cincinnati for hand rotations, Ft. lauderdale for foot/ankle, Sports in Arizona or California, Trauma in Baltimore, Spine at Mayo clinic, Joints at Cleveland Clinic......the possibilities are endless. If you are looking to do a fellowship, this is the program for you.

Cons: So your in Toledo which is not far from bankrupt Detroit but there are some nice cheap places to live about 15 miles from the hospital ghetto. The way I see it, residency should kick your ass so you can be prepared for anything and everything. The 4th year more than makes up for the rest!!!!!!!

Thanks
 
Grandview Hospital Dayton, Ohio "We train our residents how to work and how to cut"



This is one of the older school orthopedic residencies with a lot of history behind it. The orthopedic staff are a big draw for many because they are very well trained and offer almost every subspecialty. Many of the attendings on staff are grandview graduates that went on to do fellowships. Dr. Bamberger is PD and is the mastermind behind the show. He and the assistant PD are hand fellowship trained, which makes this program a hand-heavy residency, obviously a good thing if you know you want to go into hand as they have great connections to hand fellowships including an in-house fellowship. They have daily resident run didactics that typically involves the 2's giving presentations. When I was there the didactics seemed like random topics instead of catering to the OITEs. The operating experience geared more to the upper levels, where it seems like the 5's ran the show and operated all day. They boast on having a residency that produces very well trained surgeons compared to the other programs, which is somewhat true. However, in order to get to the chief year they have wait their turn in the OR as junior residents. If you're good at following orders and taking care of a service for your first 2-3 years, it's a good program. There is research potential here, but wasn't really emphasized as with most DO programs. The lifestyle here is not the best. The first six months of PGY2 year is hell where they go Q3 and run all the attendings' services; it's like their own ortho initiation. If you get an opportunity to go here, you will work very hard for the next 5 years. The attendings expect a lot out of the residents (Some much more than others), which sometimes puts a lot of pressure on them. The "malignant" atmosphere of this program has been recognized and is pretty much gone. However, some of the attendings expectations of the residents still keeps the program sometimes a little high stress. If you function well on stress, this will be a good place to go. The resident class seemed to work well as a team, but when work was done they would leave and it seemed they didn't hang out outside of the hospital. Location is decent with nice suburbs surrounding the dayton area. As a medical student, they didn't expect much from as in terms of knowledge; however, I learned more about clinic type orthopedic knowledge than I ever did which was pretty cool. The strengths are hand and joints while their weakness would be little trauma (they rotate at Florida institute for trauma which I hear is a good rotation). Overall, if you're good at following directions, like a more "professional" atmosphere, can tolerate high stress and PA duties at the junior level, it's a great program. You will come out of this program a good surgeon that can cut if you can bear the pressured 5 years.
 
Mercy St. Vincent Medical Center - Toledo, Ohio



"The Jaeblon Show"



General: This program is based in St. Vincent Medical Center a 500ish level 1 trauma hospital is the ghetto part of toledo. They are a very trauma heavy residency that boasts resident autonomy and strong didactics. Dr. Jaeblon is their PD, and may have been the most involved PD with the residents and medical students I've ever seen. As other reviews have stated, he really treats and jokes with the residents as if they're his sons. They take three a year and like high board scores, but the recent years have shifted this mentality to be more catered towards picking hard working applicants in addition to a decent board score. One of their biggest selling points is that they spend their 4th year completely out rotations which allows them to make connections for fellowships.



Attendings: In the past years they've lost some attendings including Dr. Jaeblon's son to LSU who was an absolute baller surgeon and teacher. Dr. Jaeblon runs the show and lives and breathes this residency. He was the most down to earth PD you'll meet on the trail and he'll be the first to tell you that the only reason he still is an attending on staff is for his residents. You will spend a ton of time shooting the **** with him as the residents are doing his cases. They have a couple more attendings which are all very nice and let the residents do their cases.



Residents: All laid back and nice guys, where the seniors come in every day and keep everyone working hard. You can tell though they're hard working and read their ortho.



Didactics: Toledo and Harrisburg had the strongest didactics. They even have one of their attendings video conference them to pimp them on Miller Review. Since they don't have too much case load, which I'll further talk about, you will have a lot of time to read during this residency as a junior resident. As far as ortho knowledge/didactics I still think Harrisburg had them beat.



Operative Experience: They have two main services which is essentially a trauma service then a general ortho service(which has trauma cases in it too). What distinguishes this residency is that the way cases are assigned is sometimes a free for all where seniors pick cases they want and the other ones are given to the junior residents. Also if a resident sees a patient in their resident run clinic on monday, that resident gets to do the case. Dr. Jaeblon let's the residents fly during any of his cases. This program emphasizes autonomy at an early resident level which is unlike Grandview. They emphasize doing quality of cases vs quantity. Their weak points are low case load and often times case variety. Often times residents are double and triple scrubbed, which doesn't afford the med students a ton of time to scrub in as well. Although they do get to do a ton, there are not many elective procedures done at their institution where also a majority of the operations being trauma cases. Even though they round early, they often times finish all of their cases for the day early which gives plenty of time to read as a resident or medical student.



Clinic: Very good because regardless of their resident level, if they see the patient they can book the case and do it.



Research: Wasn't talked about.



Lifestyle: Unlike the other reviews I read, i found their lifestyle pretty tolerable. The only thing that's different is that even as a senior, they're still going in early and working hard. Toledo isn't the best city but most residents live 20 minutes away. They do cover a level 1 trauma hospital so when you're on call you typically see some crazy stuff.

Pros:
1. Operative Autonomy
2. Strong Didactics
3. 4th year spent on out rotations for opportunities
4. Good place if you want to go into Trauma

Cons:
1. Case Load
2. Case Variety is a little too trauma heavy

I think this program was probably the best osteopathic program a couple years back but not anymore. They have lost some good attendings in the recent years, and i'm not sure how long Dr. Jaeblon will still be there. I would suggest only doing 2 weeks there or not rotating at all.
 
Doctor's Hospital - Columbus, Ohio

General: Despite anything I say in this review, you really can't beat living in Columbus Ohio for 5 years. This is arguably the best residency in terms of location and to top it all off, all subspecialties are in Columbus so you won't have to travel a lot if you have kids. They operate in Doctor's hospital, which is their community hospital and at Grant Medical Center, and Nationwide Children's.

Attendings: Dr. Baker is the PD, and he's a nice guy that always let's the residents do his cases over at Doctors. Their trauma attendings really steal the show and are involved during their Wednesday trauma conferences. Almost all their attendings are fellowship trained and very good at their respective fields. Resident Autonomy is very attending dependent where some don't let the residents do any of the operating.

Residents: The residents here are all well read, laid back, and nice people. It seemed their theme this year was making sure their applicants were good people in addition to the standard ortho stuff. Since it is a bigger program, you may not get that tight-knit feel that you would get at the other programs.

Didactics: As a medical student, their wednesday education is very entertaining starting with fracture conference that puts the interns in front of everyone while getting pimped by the trauma attendings. I personally didn't like their didactics where they did everything on wednesday for 5-6 hours in the morning. However, with all their residents at different places all over columbus it's really the only way they can do it. They're trauma didactics are the best, but everything else is just alright.

Operative Experience: This program is very similar to most MD programs, where their 1's and 2's will be doing more floor work while most of the operative experience comes in the senior year. They get quality exposure to all subspecialties with reputable attendings, but operative autonomy is weak. This is a "see much, do little" type of program, which is not necessary a bad thing as the attendings are very good and explain a ton during their operations. An example is that although they operate out of the one of the biggest level 1 trauma hospital in Ohio, they have fellows that are always involved in the cases. From what I observed, the residents here did not have the surgical skills as some of the other programs.

Clinic Experience: Decent clinic exposure with attendings. They do a lot of clinic on their subspecialty rotations

Research/Fellowship opportunities; One of their strongest points. This residency is now focused on putting out quality research. Because of their association with Grant and Children's they have one of the best research opportunities with great attendings. They also have more free time in their upper years that affords them the time to do it as well. This residency is very well known for churning out great fellowship matches.

Lifestyle: After you get past intern year and your 2nd year, this residency offers one of the best lifestyles for orthopedics that I've seen. When you're not on trauma, you're on subspecialty services where you essentially are on the same schedule as the attending. They have a night float system at doctor's so when you're done with your service for the day, you can just go home. Good lifestyle + Great city makes for a pretty decent 5 years.

Pros:
1. Location and everything is in-house
2. Outstanding subspecialty exposure at high quality institutions like Grant Medical Center and Nationwide Childrens
3. Good fellowship placement
4. Great research Opportunities
5. Excellent Lifestyle

Cons:
1. Operative Skill and autonomy

This is an excellent program, but didn't fit what I was looking for.
 
Pinnacle Health - Harrisburg, Pennsylvania

"We are number 1"

General/attendings: For the most part look at jbru3423's review because it's spot on. I'll try to add more, but everything is basically said; this is a top tier ortho residency. This is an elective heavy community program located in the capital of pennsylvania. You will be surprised how high volume their community hospital cases are where they can have 3-5 attendings operating a day with the last case ending anywhere from 4pm-9pm. Joints here are their biggest strength, which also includes tons of joint revisions where you don't see everywhere. Many of their attendings are general surgeons, however they all are very experienced doing case variety. If you're scheduled with Dr. Deluca, your cases can range from an ACDF, rotator cuff tear, joint replacement, 2 body fusion, to a hip arthroscopy all in one day. You will come out a great community surgeon with the variety you see from their core staff. They now will cover 3 Pinnacle Health hospitals, and the ortho floors at osteo and downtown are nice as hell too.

Residents: All residents here are very bright, hardworking, and want to be the best. It's a great friendly competitive atmosphere where they expect a lot from each other. As a med student, you'll notice the hierarchal atmosphere where the interns and juniors get pimped during morning rounds, but it's never malignant. They are all a great bunch of guys that are just all so well read in ortho. If you are lucky to be considered for a spot here, you will be working with all A game residents that want to be the best and expect the best.

Didactics: They had the best didactics on the ortho trail. They have some form of didactics everyday including OITE review, OKU, journals, fracture conference, pre-op conference, you name it. I came into this rotation having Netter's, Handbook of fracture, and chapters of millers memorized cold, and I still learned tons and just tons of ortho during the month. I would argue their interns know more textbook ortho than many 2's and 3's at other places.

Operative Experience: The residents are always in the OR similar to Oklahoma City and Fort Lauderdale's program. They typically cover 2 attendings a month so it's similar to a mentorship program except you get double the OR time. You will graduate logging in a ton of cases. These guys know how to operate.

Clinic: Resident run clinic so it's good.

Research: Not much research opportunity here. This program has its time emphasized on OR exposure and ortho didactics. Most of their residents are technically strong enough to purposely opt out of fellowship opportunities and go right into general ortho.

Location/Lifestyle: Harrisburg is a decent city to live in. It's somewhat close to Philadelphia and Baltimore. A little further out there's NYC and DC. They get pretty well compensated as residents too. Lifestyle wise, you will work very hard here. When you are finished with your work in the hospital, you're expected to stay on top of their rigorous didactic schedule.

Med Student Experience: This is the orthopedic program to rotate if you want to show your ortho knowledge. The ortho residents pimp the med students decently, which helps you because you learn, and it helps show them who knows their stuff. This was a great rotation where you function as a sub-intern; you do all the progress notes on your patients, pre-op, pre-write scripts, post op, and scrub into 5-8 cases a day. They give you controlled responsibility, which is getting more rare on the audition rotation these days. Know your netters and handbook of fractures, and you will shine.

Pros:
1. Operative Case Load
2. Didactics
3. Residents

Cons:
This is an all around strong community program, and I feel it's definitely the best orthopedic program on the East Coast. This program is a little bit weaker if you're looking for research and fellowship opportunities. They are also an elective heavy community program so they don't get as much trauma training as other places, which is not that big of a deal depending on your goals. I think they only get a few elective rotations as well since they have to cover their 3 hospitals. You will work hard here.

During the 2013 season, I rotated through St. Joe's, Doctors, Grandview, Toledo, Harrisburg, and 1 MD program. All the programs I went to would have made me come out of their program a decent surgeon. Harrisburg and St. Joe's Warren,Ohio were the top 2 osteopathic ortho programs for me.
 
Wozo30
Joined:
Nov 21, 2008
Messages:
40
Status:
Medical Student
Mercy St. Vincent Osteopathic Ortho

I checked my email this afternoon and found out I matched ortho in Toledo, OH. I am SOOOO stoked about this place I had to write my review because the previous review had some this wrong and I wanted to elaborate on a few points which I felt made this program unique for me.

Program: Mercy St. Vincent in Toledo, OH

General: Program is solid. Director is Jaeblon - has been PD for over 30 years. He tells it how it is and he makes sure his residents are prepared for anything after graduating. They take 3 a year and only choose from residents that rotate here, but the key is you have to have the board scores in order to even rotate (at least a 600). Residents get hands on during their first year and continue to do so as they move to 2nd, 3rd year and so on. This is the only osteo ortho program at a Level 1 trauma hospital.

Attendings: Residents book cases out of clinic with Jaeblon. The younger residents do the majority of the cases with the senior resident guiding them along the way. Dr. Jaeblon sits in the OR and lets the residents problem-solve through the case. The new trauma guy does a decent amount of cases (long bone fractures, pelvis & acetabulum, proximal humerus, nonunions). They have a fellowship trained spine surgeon who does some general as well. There are 2 fellowship trained sports guys (one guy just started a few months ago) who operate at different hospitals/surgery centers around town with the residents. 3 other general orthopods round out the main operating staff - 2 of them operate at other hospitals with resident coverage (students could not go to these hospitals). 3 Hand/Plastic surgeons take all hand call at St. V's and residents are a big part of those cases as well. 3 pediatric orthopods are located across town at another hospital where a pgy-3 spends 4 months (I felt this was a big plus since you did not have to leave town). One of the chief residents will be joining the pedipods after fellowship. In general, all the attendings teach the junior residents during cases and let them operate very early compared to other programs.

Residents: A big part of the program is senior residents teaching the junior residents. The 3 senior residents were all very well read (1 is doing a hand fellowship, 1 a total joints fellowship, and 1 a pediatrics fellowship). The senior residents run morning didactics and keep the juniors on their toes with pimping during cases. PGY-2s and 3s take the majority of the call with PGY-1s on for 8 shifts a month. PGY-5s take call from home and come in if needed. All of the residents get along well and like to have fun - they frequently get together outside the hospital. Residents also cover local high school sports on Fridays.

Didactics: Usually something every morning:
Mondays: Miller review and Topic lectures by Dr. jaeblon
Tuesdays: sports articles
Wednesdays: OITE review followed by a PGY-1 lecture
Thursdays: Hand lecture, Trauma conference, resident lecture, and articles with Dr. Jaeblon
Friday: Cadaver presentations
Usually a sawbones gathering or evening lecture at least once a month
Every resident presents their yearly paper in a power point lecture every month

Operative Experience: Very early operative experience with seniors teaching juniors. A lot of trauma cases in the summer - good amount of sports cases - spine cases 2 days a week (1 day at St. Vincents and usually another day at another hospital) - hand cases weekly. There are no assigned rotations - the residents divvy up the weeks cases in clinic every Monday (first case is a total hip, next case could be a spine, then Carpal tunnel, then a tibia nail that came into the trauma bay earlier that morning).

Clinic Experience: Mondays all day - clinic is clinic - 40-50 patients - residents book a lot of cases staffed by Dr. Jaeblon. Lots of total joints, shoulder and knee scopes, and hand cases.

Research: Not much of a presence right now. The new sports guy who just started is lining up research projects with the junior residents. He also is in charge of making sure the yearly paper requirements and power point presentations are complete. Most of the residents submit their papers to ortho journals.

Lifestyle: These guys are very busy, well read, and can handle any trauma that comes in the door. Residents work hard but also know how to have fun. Lots of traveling to different hospitals in town - 6 in Toledo and 1 in Defiance.

Pros: the biggest draw to this program is the 4th year. This year is spent on out rotations where residents set up rotations in areas they don't get a lot of (like foot/ankle and Tumor). THE ENITRE YEAR IS SPENT OUTSIDE TOLEDO!!!!!!!!! Residents usually rotate in areas that they want to do a fellowship to get exposure. Many of the residents go to places where family and friends live to do their rotations. A lot of the guys went to Cincinnati for hand rotations, Ft. lauderdale for foot/ankle, Sports in Arizona or California, Trauma in Baltimore, Spine at Mayo clinic, Joints at Cleveland Clinic......the possibilities are endless. If you are looking to do a fellowship, this is the program for you.

Cons: So your in Toledo which is not far from bankrupt Detroit but there are some nice cheap places to live about 15 miles from the hospital ghetto. The way I see it, residency should kick your ass so you can be prepared for anything and everything. The 4th year more than makes up for the rest!!!!!!!

Thanks

Not the only program at Level 1 trauma center. Riverside, York, Wellmont, and Broward are all level 1 Trauma centers. Doctors is connected to Grant which is also level 1 trauma center. Ingham covers Sparrow which is also level 1 trauma center. Also, the program recently has had some concerns of malignancy with one resident resigning.... that's almost unheard of at any ortho programs. In addition, past 3/4 matches, they have ended up in scramble. They play a game where they keep in touch with numerous candidates early on, leading them on, and then only rank 3 guys who verbally commit to them. This is a very bizarre way to rank candidates, in addition, they are ridiculously obsessed with boards and neglect guys with solid boards (630-650) for guys that have 700+, even if they were hardworkers.

In my experience, this is a solid program, but they need to do a better job recruiting residents. Also Jaeblon is really old and the program has no direction as to what will happen once he will retire. It also very bothersome to find out that one of their PGY3s quit middle of the residency.
 
  • Like
Reactions: 1 user
Anonymous

Botsford General Hospital

Attendings/PD: There are six primary attendings. One is a trauma guy who is new. (Dr Colen was their old trauma doc but died unexpectantly in July 2012. They say the program took a big hit, which seems true). The new trauma doc is not nearly as busy and splits time between other hospitals. The other 5 attendings all trained at Botsford – ie very inbred program. Three are fellowship trained (Hand, F&A, Spine), the rest are general. All attendings are older (55+), meaning they are nearing the end of their careers and it shows. Appear to be pretty inflexible to learning new techniques or trying new methods. All have kind of “different” personalities. All attendings are very handsy and didn’t seem to let the residents do much. Dr Linard is the PD --- older guy who has been there forever. Cusses out pts/residents/staff as his way of joking around, but was over the top. Saw him throw stuff across the OR bc he was pissed, and it didn’t seem like that was a rare occurrence.

Didactics/Teaching: Resident led, which is true at all DO programs. However, no attending involvement here. Don’t even attend Fx conference. Residents will say this is the strongest part of their program (and believe it’s stronger than some other programs). Have morning lecture/fx conf every day for 30min. No dedicated OITE or education review. Most residents seem smart/well read, but don’t appear to use textbooks like Rockwood, Skeletal Trauma, Campbells, etc. Rather, use articles instead. Some residents are a little cocky about their knowledge (ie quote tons of articles), but it was all based on what they’ve read, not anything they’ve actually seen or done in the OR.

OR Experience: Poor. Poor autonomy and no dedicated OR time for residents until third year. First two years are spent on floor or in the office. Only time you see the OR first two years is if you’re lucky enough to be on call and cases come in (and don’t get stolen by seniors). Felt like seniors skills were sub-par compared to other programs. Additionally, attendings are very handsy and don’t let the residents do much. Never saw a senior do a skin-to-skin case. As a junior you are most always paired with a senior – lots of double scrubbing.

Clinical Experience: Good, if you like clinic. PGY-2’s spend every third month on clinic, and then some. Seniors still going to clinic.

Research Opportunities: None that I heard of. All residents have to publish/submit some sort of paper. Some residents are trying to start up research projects.

Residents: Best aspect of the program. Seem smart, although some a little arrogant with their knowledge. Get along well and hang out outside of work.

Location/Housing: Not sure about Farmington Hills, but cool places 15-20min away (ie Royal Oak, Ferndale).

Limitations: OR experience, and exposure to variety of cases in general. No complex revisions, or cases in general (multi-lig knees, etc), done here. Also, attending involvement is very poor, and the attendings would be tough to work with. They basically treat the residents like PA’s and not much more (limited teaching, OR autonomy, etc).

Other: Is a level 2 trauma center, so get a decent amount of trauma but didn’t see much general trauma while I was there (ie hip fxs, distal radius, tibia, etc). Seemed that by not having Dr Colen anymore, the program had a big hit. Sounded like nearly 1/3 of the residents came to the program simply because of him. Lots of spine trauma, and spine in general. Family friendly program. Only program I know of that gives post call days, and residents love it, but seems to breed a somewhat lazy environment.

Pros:
1. family friendly lifestyle
2. resident camaraderie

Cons:
1. operative experience
2. attendings
3. case volume

Overall Experience/Conclusion: Residents will tell you this is “the best ortho program in MI”, and that may have been the case a while back, but it doesn’t seem like anything has been done to improve the program in the last 10yrs. There is a very pompous attitude about Botsford overall (from all programs, not just ortho). They supposedly used to call it “Botsford University”, and while I don’t doubt that there is good training and maybe it used to be a top DO spot, it doesn’t seem like they have made many improvements lately. Would be concerned about where the program is headed in the next 5+ years as many of the attendings, if not all, will be slowing down and retiring soon, and they haven’t added anyone new. In hindsight, would not rotate here again and would recommend looking elsewhere for your ortho residency. Heard that intern year here is brutal.



Henry Ford Macomb, Detroit, MI

Staff/Faculty/Chairman: Dr Fremont Scott is the PD. Nice, knowledgeable, somewhat hands-off with students. About 5 primary orthopods involved in the program, most are general, but take call with all of the docs at the hospital (22). As a result they get a lot of good general call (level 2). Three others loosely involved with the program – one general, but does tons of UE; one total jts; one sports. There is a ton of orthopedics to be done here and lots of fellowship trained guys not officially associated with the program. All the attendings love to teach and are great to work with.

Didactics/Teaching: Good, strong emphasis in the program. 6:30-8am M&F, and then go downtown on Wednesday’s from 7a-12pm to attend lectures with the MD program.

Operating Experience: Their volume is not as high as some other programs, but the autonomy is excellent. They get into the OR early (interns doing hemi’s) and are given free reign by attendings. Some double scrubbing, but mostly just on trauma cases.

Clinical Experience: Mostly as a 2 and 3. Depends on the rotation or service you’re on. Not too much clinic, but enough to know when not to operate (important).

Research Opportunities: HF main seems to be a decent resource for this. Many of the seniors had good projects going and were presenting at national conferences.

Residents: Take 2 residents/year. Great group – nice, smart, willing to teach, but not cocky. Help each other out if someone needs to leave early or something. Very laid back, but still efficient and get their work done. Most married and/or with kids.

Location/Housing: Metro Detroit area…some residents live close by, others live in Royal Oak (great area). Most have bought a house.

Limitations: Henry Ford does not allow the residents to do out rotations, which can be a negative when/if looking for fellowships. But the PD recognizes this and seems to be willing to work the system. Volume can seem a little low, but they also only take 2 residents/year and there is plenty of call cases to go around. NO food for residents.

Pros:
1. operative autonomy
2. didactics
3. lifestyle

Cons:
1. volume
2. limit on out rotations

Overall Experience/Conclusion: Excellent. Very laid back program that wants to graduate competent and proficient surgeons. Definitely more of a lifestyle/family friendly program (often done at 3 or 4pm), but doesn’t seem to affect their knowledge or OR skills. The program recently moved its base to HF Macomb and as a result they are much busier, and there seems to be a lot of opportunities to do rotations with the other orthopods at the hospital (jts, hand, FA, etc). It seems that the relationships and opportunities with those docs will continue to grow over the next few years and the operative experience and volume should only get better. Would rate 8/10.


 
Anonymous

Program: Mclaren-Oakland, Pontiac, Michigan

General: Base hospital is a level 2 trauma center, about 300 beds, and is located in Pontiac, within metro Detroit. The city of Pontiac is pretty run down, but the hospital itself is nice. Majority of the residents at the hospital live in the surrounding suburbs and commute into Pontiac. They take 3 residents per year.

This program is different from the other programs that I rotated through in that they have a lot of out rotations. Six months of ortho during intern year is split between Mclaren-Oakland and Huron Valley Sinai Hospital. Second year is mostly spent at Mclaren-Oakland covering all the ortho that comes to the hospital, working with the attendings listed below as well as doing spine. Third year is mostly out rotations at DMC and Childrens Hospital of Michigan. Fourth year consists of more out rotations including 4 months of trauma at UMC in Las Vegas. Fifth year consists of electives and covering some of the attendings at Mclaren-Oakland.

Attendings: Dr. Fugle is the PD and has been for over 40 years. He is a general orthopod, but can and will operate on everything. He loves to joke around and have fun while he works. He loves hard working students and residents as well. Dr. Nallamothu is another general orthopod who runs a very busy practice. He is an excellent surgeon, very knowledgable, and great to work with as a student and a resident. Dr. Ciarlone and Dr. Limbert are also general orthopods who are good to work with, though not as busy as Nallamothu. Every attending has a variety of cases from joints to hand to trauma to scopes. Dr. Prince is fellowship trained in sports and runs a busy, successful, well respected multi-specialty practice. He is great with knees and shoulders and is good to work with.

These are the attendings that you'll work with as a student rotating at Mclaren-Oakland, however these are just a few of the attendings that are affiliated with the program. These are also the attendings that will be present during your interview and are the ones you want to impress while on service. There are many other attendings that you'll rotate with as a resident that will be at different hospitals. As a resident you work with some big names in the Detroit Metro area, including Dr. Lemos who is the team physician for the Detroit Tigers.

Residents: Best group of residents that I was around. I personally felt that the residents at this program were the most cohesive group compared to other programs. Everyone got along well with each other and they were always helping each other. No malignant personalities present. They were all great friends inside of the hospital and outside of the hospital. Furthermore, I felt like they were very knowledgable and well read on orthopedics.

4 seniors this year, 2 are going on to do a sports fellowship (1 at Mayo Clinic in MN and 1 with Dr. James Andrews in Birmingham, AL), the other 2 are going on to do a joints fellowship (1 in Florida and 1 at Indiana Univ.).

Didactics: Didactics on Tuesday-Friday. They are run by the senior residents so they may differ each year. They typically consist of lectures given by the 2's and the seniors and also questions for OITE. Tuesday's are always fracture conference, where they go over the trauma that came in over the past week.

This isn't a strong point of the program, however, I thought it was better than the didactics at the other programs I rotated through.

Operative Experience: Hands down this program had the best surgical residents that I saw. They get very early operative experience and it shows. The seniors were usually doing most of the surgeries on their own and even let me do stuff in the surgeries as well. In my honest opinion, I thought that some of the 3's were better surgically than some of the seniors at other programs. This is definitely a program you want to consider if you like to get early operative experience. Some of the attendings at the hospital let the 2's do a lot of the surgery on a total hip or total knee, which is unheard of at other programs. Also, you get scope experience your first year with one of the sports attendings down at Huron Valley.

Clinic Experience: There is not a resident run clinic like at other programs. Clinic experience consists of clinic with whatever attending you are covering. If the attending you are working with has clinic on Monday and Wednesday's then you are in clinic on those days as well.

Research: Residents are required to do some sort of research project during their residency. It did not appear that research was really important at this program, however, if you are interested in research then there are definitely opportunities, especially with the connections to DMC.

Lifestyle: Good lifestyle compared to residents at other programs. It didn't seem that the residents here hated their lives. Again, great group of residents that like to have fun outside of the hospital and tend to do stuff together outside of the hospital. Ortho call starts second year and is q 3, but you are not required to stay at the hospital.

Pros: Great group of residents and attendings to work with and learn from. This is a very good program to get hands on experience as they let the residents operate early. Again, I felt the seniors here had the best operating skills. Everyone in this program is friendly and would be great colleagues. This program is also great in that it allows you to make great connections throughout your training if you're interested in doing a fellowship. This program has seen its graduates get into some stellar fellowships in the past. If you're not interested in doing a fellowship, this program prepares you well to be a generalist. The previous seniors all went into general practice and are doing well for themselves.

The 4 month trauma rotation at UMC in Las Vegas is a definite plus for this program. The residents speak very highly of the rotation and say you see and learn a ton, also, it's Vegas.

Everyone at the hospital is very nice as well. I felt that as a student, the staff and even other residents were very friendly and helpful. It definitely was not expected.

Cons: Location - Pontiac isn't exactly paradise and neither is Detroit. There are a lot of nice suburbs around metro Detroit though and there's always something to do it seems. The weather is kind of a bummer too, not necessarily the cold, but the lack of sunshine during the winter.

The number of out rotations can be a negative depending on how you see it. All the hospitals are in metro Detroit and it's not like you'll live in Pontiac so commuting will be the norm. However, the 4 month stent in Las Vegas during your fourth year can be a drag if you have kids.

I rotated through Lansing and Grandview in Dayton and overall, I thought Pontiac was the best program. I was pleasantly surprised at this, especially because there is little to no information on this program compared to what you see about some of the other programs. This program is definitely a program that you'll want to check out as I learned a lot from the residents and attendings and as a student I was able to help out in the OR as opposed to just standing, watching, and retracting.
 
Program Name: McLaren Greater Lansing, MSU

General: I think this program is a little under the radar compared to the other "top DO ortho programs" (ie Doctors or Harrisburg), but having rotated at both I can say this program is at least on par, if not better, in all aspects. They have all subspecialties covered except tumor, which is unheard of in the DO ortho world. Peds is 3mo in Cincinatti 4th year, and also just added a peds doc in Lansing so may have option to do it in town in the future. Trauma is 3mo at Ortho Indy in 4th year, although they get a ton of trauma through Level 1 Sparrow Hospital (and may have option to do it there in the future as well). Excellent fellowship placement: Cleveland clinic jts, Mayo for onc, Sports with Andrews, Sports at Beaumont, etc. Additionally, lots of attendings come back to work with the program, or there are others with fellowships, so the possibilities for connections are very good. Not too board heavy – have guys from low 500–700+. The residents pick who gets an interview and then the attendings pick who gets in. Thus, must rotate here and would recommend a full 4wks. Had multiple residents say what they look for is “someone who works hard, improves, and that I would want to get a beer with”. Can “moonlight” at the ortho hospital starting mid-2nd year and make $400/night.

Attendings: Dr Dietzel is the PD. Quiet, but once you spend time with him he’ll joke with you and seems like a great teacher. Pts love him. Dr Taunt and Dr Cochran are total joint machines. Both also do a lot of trauma and are very vocal in the program and selection process. Too many other attendings to list. Most are younger and passionate about resident education. Majority are fellowship trained, which is a huge plus (and really a rarity in a DO program). Also work with trauma guys at Sparrow, which is Level 1. Dr Swords, F&A at Harborview, isn’t covered by the residents but does a ton of trauma and will let you scrub with him over the weekends he’s on call. Seems like an incredible experience. All sports guys cover MSU sports (Dietzel is football team doc). Overall, no malignant attendings and all are there to teach.

Didactics/Teaching: Strong - best didactics I saw. Hospitals are close enough that everyone is able to meet for education each morning. Good mix of pimping, but not shaming you into knowing the material. All the residents are smart and it is just understood that you’ll read and know your stuff. Morning lectures, Fx conference, journal club, weekly OITE, etc. Regularly use skeletal trauma, campbells, journals, etc – not just review books or material like a lot of other programs. It’s evident that they are there to learn ortho, not just pass the OITE. *Unique here is that approximately 1x/week they have a specialty conference where Taunt (jts), Flood (spine), or Truluck (hand) will attend and the resident who is on their service gives a lecture and they chime in. Very good, very informative, and I’m guessing a rarity compared to most programs (didn’t see it anywhere else).

Operating Experience: Excellent. Operate quite a bit. Starting as a PGY2 it is service based. Some services you only cover that doc and go to the OR and clinic, but others you cover two docs and are in the OR every day. Either way, as a PGY2 you’re the only one on service – this means no cases are stolen by seniors and no double scrubbing ever. The level of autonomy varies with each attending, but the seniors appeared confident and skilled. One senior going into general, one to Hand/UE, one to joints. One of the few true orthopedic hospitals left, which is nice. Also, lots of complex cases here (multi-lig knees, complex revisions, etc) - not just bread and butter ortho.

Clinical Experience: Depending on the attending and service you’re in the clinic at most 2x/week. PGY2’s cover resident clinic 1x/week, Wednesday afternoons. Never went but heard it’s fine. Seemed like it was a good amount of clinic to help you learn, but not too much.

Research Opportunities: Have a research lab in the same building, but the residents are too busy to do any real research. However, opportunity is there if you want it. Required to publish a paper years 3-5.

Residents: Take 3-4/yr. Awesome group. Get along very well and have each others backs. Seem like they’re all brothers (there are two girls, but they get along well and the guys include them on everything). Nobody is malignant or too egotistical. They like to teach and quiz, but it’s a good balance. You can tell they hand select their residents because they all work hard, are chill, and smart.

Lifestyle: Lansing kinda a hole, but there are a lot of suburbs around that are nice. Downtown Lansing is ok, but not really much too it. Biggest draw seems to be East Lansing and MSU. Lots of surrounding lakes, beaches on Lake MI, hiking, etc nearby that the residents and attendings go to. Has plenty to keep you busy with the limited time you’ll have. Also, some of the attendings are MSU docs so residents often get tickets to MSU sporting events. Residents say “we’re busy”, which is true, but not overwhelmingly so. They work hard, but also have some chill days depending on the service. Location is probably the only limitation, but it’s doable.

Pros:
1. Operative time/experience
2. Residents camaraderie
3. Didactics
4. Fellowship placement/opportunities
5. Attendings

Cons:

1. Location
2. Not “lifestyle friendly”
 
Anonymous

On the topic of meadowlands and Plainview:

Meadowlands as of now is a low tier residency. As of now, you will not become a good surgeon (or even an average surgeon) coming out of it. Rotations are up in the air for their upper levels too. Very disorganized. Would highly recommend only going there if you are not a strong candidate and need a "safety" spot.

Plainview is okay. The residents don't get to operate much. They have to do a lot of watching especially when they rotate through LIJ. I know a core student that had a spot there but really did not like it and matched instead. Their only saving grace is that they get good fellowship matches due to the attendings they rotate with even though they don't get to do much aside from retract. They interview late and most students that are competitive applicants have already verbally committed to other programs voluntarily. This year I know of 3 students that matched there because they couldn't match anywhere else.
 
Thanks for the response, sounds pretty lame... So thoughts on those vs the other jersey residencies then? I know Rowan just skipped a year of interns, and inspira? Basically number one is being a great surgeon, but I also have family in the NE and it'll be much much easier on my wife and I when we decide to ruin our lives and have kids if we have family support around. I only have 3 possible auditions, 1 is dedicated to pcom. I was thinking Rowan/inspira before, but then saw plain view takes 6 which increases chances big time. Consider myself a good applicant (630s, adequate leadership, some research, top 15% of class, athletic guy, personable..haha) (is that good? I feel like my entire class wants Ortho, and is smart too) bag those north jersey spots? (Where my wife is from, most free support up there) THANK YOU!!
 
I honestly don't know anything about the majority of the NE programs outside of hearing great things about Harrisburg. Lots of OR time and knowledgable residents from what I hear. If the anonymous poster replies to your question in my inbox I'll post their anonymous reply, or perhaps others will see your post.
 
HI, i have some questions about the MI programs. Im originally from ohio but am going to school in the northeast. I want to come back to OH or MI but have only a few electives. Could you provide me some information on the schools below? Which of the programs do you think is the best and would i have the best chance of matching to.

McLaren Oakland, Pontiac MI
McLaren Greater Lansing, Lansing MI
Metro Health Hospital, Wyoming MI
Henry Ford, MI
Genesys, MI.

Also, does anyone have information on UH regional hospitals or Summa?

Thanks!
 
From Anonymous

Program: -Largo Medical Center---Largo, FL
General: Welcome to sunny Florida. This is a great program. Underrated because it's relatively new. Will be taking 2 residents per year from now on. No rotation as a resident will be further than 30 mins from the base Largo Medical Center which is in the greater Tampa area, except for 1 (Peds). They cap their rotating medical students.
Attendings: Just added another attending starting next year and have had an influx of young talent coming in the last couple years. Program Director (Dr. Harker) will operate on anything and is top notch. Dr. Wood (also operates on anything) is one of those old time DO’s that’ll say and do anything. Both will let residents do entire case, no matter what year you are, if you’re prepared, you’ll do it. Dr. Scott and Dr. MacLaren are the sports docs who are in a partnership with Tim Tebow and Chipper Jones and deal with a lot of professional athletes. The upper extremity doc is Dr. Marcott who did his hand fellowship at Harvard. Those are the main ones you will work with during audition rotations. Many more but you will rarely work with them unless you are a resident.
Residents: Residents are top notch. PGY 5 just got accepted to the Dr. James Andrews sports fellowship. The 2 PGY 4’s have been accepted to Spine and Joint fellowships respectively. They operate a lot, and don’t necessarily work INSANE hours. The attendings are good about letting them get hands on experience. Residents feel comfortable letting students do a lot because they’re confident in their own abilities. Fun group, a lot of characters and everyone is always joking. Single and married/engaged. They get along well. There are no females currently in the program. Normally one resident per attending. Match 2 students per year.
Didactics: Didactics are Monday morning. Fracture conference from 630-800. Lectures and OITE questions from 8-12. Probably the weakest part of the program is the lecture series starting at 8. Nothing out of this world. Good news is they do OITE questions.
Operative Experience: As a student, especially with the program director you will get unbelievable OR experience and face time. Residents aren’t granted access to all the hospitals he goes too, so you will be first assist on many cases. He allows you to do 90% of IM nails for intertrochs. He allows you to dissect down on and do as much as you are comfortable doing with THA. You make cuts for your TKA’s, put in screws, close majority of the cases. Great operative experience. Best I have heard of. He is very patient and makes fun of you the whole time, but in a very benign/funny way. Saw PGY 2’s take the lead on multiple cases while PGY 5 was 1st assist and attending not being scrubbed in. This happens a lot. Operate early and often as a resident. 8 months of surgery 1st year (6 being Ortho). Many other programs only do 3-4 Ortho. Residents are very prepared and have great hands.
Clinic Experience: Each attending has their outpatient clinic that resident will go to. Nothing major, normal clinic. Once or twice a week depending on what service you are on. This is big for students to get face time with attendings. Attendings speak with students quite a bit, and residents do a good job of not getting in the way of that.
Research: Research is not a big deal at this program. They did not mention it much. Resident’s not very involved in it.
Lifestyle: Call is taken at home, and is not crazy at all. May get 1 or 2 consults, but also may get none. Many times the consults can wait until the next morning, if you even get one. Tampa is 25 mins from the hospital, and Clearwater beach is 5 mins away. It’s Florida, which says a lot in itself.
Pros: 1. Able to operate early and often.
2. Very little Scut Work
3. Time to prepare/read
4. State of Florida/Lifestyle
5. A lot of bread and butter Ortho
6. Great fellowship opportunities
Cons: 1. Lecture series Monday mornings not great
2. Must do Peds rotation in Ohio for 3 months
3. Research isn’t super emphasized
 
Program: Doctor's Medical Center, MODESTO CA

General:

Attendings:

Residents:

Didactics:

Operative Experience:

Clinic Experience:

Research:

Lifestyle:

Pros:

Cons:
 
Program: Doctor's Medical Center, MODESTO CA

General: A new program (this will be their 3rd year) located in the CA central valley. Bit of a compromise here on location if you're the type of person looking for a stellar location. THE PROGRAM MORE THAN MAKES UP FOR IT. Plus you're only a couple hours in any direction from great areas in CA. San Francisco, San Jose, Sacramento, Monterey etc. Modesto has a low cost of living and is pretty family friendly depending on what area you decide to live in.
From day one on rotation you meet the attendings and they introduce themselves and shake your hand (something that didn't happen anywhere else I went). They care alot about the students' experience here and they expect the students to work and to read every single day. This is just a continuation of what they expect from their residents and it shows. They are extremely well read and they know how to operate.

Attendings: Best I worked with throughout the audition season. They have dedicated Sports, Joints, Trauma/Pelvis, Hand, attendings who all also do general bread and butter ortho stuff in addition to their specialty. They will be adding several more attendings within the next couple years. Dr. Trzeciak is the PD and is a hand guy. Cares alot about the residents' education and them getting involved in research and submitting papers but also heavily emphasizes the OR time. My first day on service I saw an intern (scrubbed with the PD) operate more than the PGY2's at other places I had been. These attending's hand over the knife more than anyone else I saw. And not in an irresponsible way. If the resident is prepared. They do the case. Best thing about it is that the attendings love to teach.

Residents: Awesome group of people to work with. They care alot about making sure that everyone is up to snuff on their education and that they're getting to operate. No malignant personalities here at all and they are willing to teach a ton. They're also willing to hand over tools in the OR to student's who show that they've done their reading and are prepared for the cases. This was also something that was rare compared to other places.

Didactics: HUGE advantage to this program. Every morning there was resident run lectures (which attendings actually came to) for an hour before going into the OR. If there was extra time at the end the senior goes down the line pimping the students and juniors. A bit stressfull with attendings in the room but it makes you better and forces you to be on your game. Formal fracture conference every monday morning with at least 4 or 5 attendings. Best fx conference I attended because they got the students involved and it was more than just "Oh read this xray". They expected us to read on treatments and what to actually do with a fracture rather than just memorize a classification.

Operative Experience: Biggest perk of this program. The residents are one to one with attendings and they are expected to be doing a significant amount, if not all of the case. When the 3s are operating the attending will scrub but usually stand back and give directions and the resident does the entire case. Obviously for more intense cases the attendings are more involved but they really care that the residents learn to operate and operate early. OR time here matches and exceeds any of the "high volume" programs out there and these guys get the knife, rather than scrubbing in to retract. My last week at the program I watched a resident 6 weeks into his PGY2 throw in a TFN with reaming and distal interlocking screw in......wait for it.....19 minutes!!! He wasn't rushing, he just knew the procedure cold.

Clinic Experience: Clinic is done in 2 places. Private clinic at the attendings' offices down the road from the hospital and also at the HSA community clinic. Some of the attendings run busier clinics than others but it's nice because you end up scrubbing in the cases that you help book. HSA county clinic is an underserved/uninsured population that has some crazy pathology. You spend a couple days here as a student and get to see how the residents run things. The attendings are there basically as guidance but the residents see everyone and book their own cases. Even the PGY1s. If they book the case they are the ones operating.

Research: PD cares alot about research and expects his resident's to always be looking out for potential papers to submit. TONS of research opportunities at this program.

Lifestyle: Pretty decent since they still have several PA's that cover some weekend and night calls. The intern year seems like the busiest but all of the residents didn't seem burnt out. Most of them had families with kids so their off time was definitely important to them.

Pros: OR time, attending's that teach and give up the knife, Resident run clinic, research, didactics. This could definitely be the best program in the country. Definitely get here to rotate if you can.

Cons: Location (depending on what you're looking for), High expectations (this was a pro for me but not for some others that I saw)
 
Thanks for the great review.. Definitely agree that intern year is rough :)
 
Can anyone offer any insights on the Ventura program? I know this is a new program but information has been difficult to come by. Looks like they are building a new hospital that should be nice, but there have been swirling rumors about Trauma case load concerns with a country lvl 1 down the street. Any info/reviews would be appreciated!
 
  • Like
Reactions: 1 user
What are your opinions concerning the newer Orthopedic Surgery programs and their viability come the ACGME merger? My concern is that some of the smaller and/or newer programs will not have the budget to make the cut and I don't want to invest into a program with a high risk of termination. What do you guys think?

Edit: Lastly, what programs give interview invites without an audition rotation? And of those programs, which have actually matched a student that did not audition? Thanks a ton!
 
Last edited:
Does anyone have information about the Tennessee program?
 
  • Like
Reactions: 1 users
It's getting to be that time of the year again. Please post your experiences with audition season. Rank lists should be in but if anyone would rather post anonymously please message me your review and I will post for you.

Program:

General:

Attendings:

Residents:

Didactics:

Operative Experience:

Clinic Experience:

Research:

Lifestyle:

Pros:

Cons:
 
I would love to hear any program descriptions from the current 4th years that finished up auditions!
 
Currently looking at a few MI programs (Genesys, Garden City, Mclaren Macomb) and am hoping that some 4th years can shed some light on these programs!
 
Any info on Samaritan in Corvallis, OR? Are they boards heavy or require any specific scores/rank to rotate or to get an interview?
 
Any info on Samaritan in Corvallis, OR? Are they boards heavy or require any specific scores/rank to rotate or to get an interview?

Based on what I've heard, I believe that they are boards heavy
 
  • Like
Reactions: 1 user
Can anyone offer any insights on the Ventura program? I know this is a new program but information has been difficult to come by. Looks like they are building a new hospital that should be nice, but there have been swirling rumors about Trauma case load concerns with a country lvl 1 down the street. Any info/reviews would be appreciated!

Unsure which exact rumors you're talking about, but I'm a resident at Ventura County Medical Center (VCMC, the original "Ventura program", unopposed family med, ACGME accredited). We are a level 2 trauma center and are the designated trauma center for this region of the county. If you're asking about the new residency programs at "Community Memorial Hospital" (CMH) also in Ventura (AOA accredited, several different smaller residency programs), they don't really take care of trauma cases - ambulances bring trauma cases to us and if patients walk in to CMH with any significant trauma they are transferred over to us (where, incidentally, Family Med residents, all of whom are certified in Advanced Trauma Life Support, run the trauma codes with attendings as backup). If you WANT experience in managing trauma, CMH is not a great place to be - though Cottage Hospital up in Santa Barbara does lots of trauma. If you want to avoid trauma, CMH could be a great fit for you.
 
  • Like
Reactions: 1 user
Anyone have a recent review of Erie's program? I've heard positive and negatives about the program, like that they lean heavily towards LECOM students, and many of their rotations are away from the base hospital. Any insight from those who matched or rotated there would be great. Thanks
 
Top